LSAT and Law School Admissions Forum

Get expert LSAT preparation and law school admissions advice from PowerScore Test Preparation.

 Administrator
PowerScore Staff
  • PowerScore Staff
  • Posts: 8917
  • Joined: Feb 02, 2011
|
#71241
Complete Question Explanation

Parallel, Sufficient and Necessary. The correct answer choice is (D).

The abstract structure of the argument is that the author makes a conclusion recommending against a course of action ("the mayor shouldn’t adopt her rival’s controversial proposal"), on the basis of two conditional premises. One of those premises states the (possible) negative consequence that follows "if" the proposal "succeeds." The other premise states the (certain) negative consequence that follows "if" the proposal "fails." Key to the argument is that the conditional statements in the premises address the two possible mutually exclusive outcomes (either success, or failure) that could follow from adoption of the proposal.

Answer choice (A): The conditional premises are the clearest sign of a mismatch here. They do not begin from mutually exclusive sufficient conditions, as the premises of the stimulus did. For example, to match the premises of the stimulus, since the first premise began from a sufficient condition that says what happens "if he stops," the second premise would have to begin from a sufficient condition that says what happens "if he does not stop."

Answer choice (B): As in answer choice A, the premises of answer choice B do not set up mutually exclusive sufficient conditions. Rather, the first premise is not a conditional statement, and does not set up what happens in one of two circumstances that could occur from Joni self-publishing the novel.

Answer choice (C): The premises in answer choice C are not conditional, and this is enough to reject it as a structural mismatch for the stimulus argument.

Answer choice (D): This is the correct answer. As in the stimulus, the conclusion recommends against a course of action (Alvin should not submit his paper). The premises state the negative consequences that follow from either of two mutually exclusive outcomes of Alvin submitting. If the paper is accepted, there's a chance of one negative consequence (not meeting a deadline). If the paper is rejected, there's a certainty of another negative consequence (Alvin's losing confidence in it altogether).

Answer choice (E): While some students might reject this answer because it makes a positive rather than a negative recommendation in the conclusion, that is not sufficient reason to reject the answer choice. Rather, there's a more subtle reason answer choice E is wrong. One of the two necessary conditions in the premises in the stimulus was certain to occur (the mayor "will get blamed"). In answer choice E, both of the necessary conditions in the premises of the argument are uncertain ("she risks not being able to go," and "she will likely still be able to negotiate"). This makes the validity of the conclusion slightly less secure than in the stimulus, and renders answer choice D a better match.
 lsathelppls
  • Posts: 5
  • Joined: Nov 12, 2018
|
#73214
I chose B, but the correct answer is D. Those were the two answers I was contemplating. Is this because the success that Joni would have found occurs second in the order of statements? In the question stem the success occurs first?
 Jeremy Press
PowerScore Staff
  • PowerScore Staff
  • Posts: 1000
  • Joined: Jun 12, 2017
|
#73242
Hi lsathelppls,

It's not the order of the premises that knocks answer choice B out of contention. Remember, the premises and conclusion of any argument can be presented in a different order (any premise or conclusion can come anywhere in the argument you want it to), and that won't change the logical relationships.

The reason answer choice B is incorrect is that the premises don't present the outcomes of two mutually exclusive possibilities. In the stimulus, we have one premise that describes a (potential) bad consequence if the proposal succeeds, and we have a second premise that describes a (definite/certain) bad consequence if the proposal fails. In answer choice B we do have a premise that describes a (definite/certain) bad consequence if Joni's self-publishing succeeds. What we're missing is a second premise that describes a (potential) bad consequence if self-publishing fails. Instead we have a second premise that describes a (potential) bad consequence (readers are unlikely to take it seriously) that happens regardless of whether the self-publishing succeeds or fails. That's a different kind of premise than the stimulus had, and makes the argument logically different.

I hope this helps!

Jeremy

Get the most out of your LSAT Prep Plus subscription.

Analyze and track your performance with our Testing and Analytics Package.